0% found this document useful (0 votes)
47 views

Solution.: D - N D - N D - N

This document provides a summary of a number theory exam containing 6 questions testing various concepts. Question 1 involves proving identities relating arithmetic functions using Dirichlet convolution and Möbius inversion. Question 2 examines properties of Dirichlet series and convolution of arithmetic functions. Question 3 proves an identity involving a multiplicative function and applies inclusion-exclusion. Questions 4 and 5 involve solving congruences and computing powers modulo a prime. Question 6 proves Euler's criterion and relates quadratic residues to primes congruent to 1 modulo 4.

Uploaded by

tonynugan
Copyright
© © All Rights Reserved
We take content rights seriously. If you suspect this is your content, claim it here.
Available Formats
Download as PDF, TXT or read online on Scribd
0% found this document useful (0 votes)
47 views

Solution.: D - N D - N D - N

This document provides a summary of a number theory exam containing 6 questions testing various concepts. Question 1 involves proving identities relating arithmetic functions using Dirichlet convolution and Möbius inversion. Question 2 examines properties of Dirichlet series and convolution of arithmetic functions. Question 3 proves an identity involving a multiplicative function and applies inclusion-exclusion. Questions 4 and 5 involve solving congruences and computing powers modulo a prime. Question 6 proves Euler's criterion and relates quadratic residues to primes congruent to 1 modulo 4.

Uploaded by

tonynugan
Copyright
© © All Rights Reserved
We take content rights seriously. If you suspect this is your content, claim it here.
Available Formats
Download as PDF, TXT or read online on Scribd
You are on page 1/ 5

MAS 5215/001 Number Theory, Spring 2015

Test 3 (60 minutes)


Date: 04/23/2015
Name:
Show ALL steps.
(1 pt for writing your name nicely)
50 points equal 100%.
Question 1. (5+5 points) Assume that f, F are
P arithmetic functions.P
(a) Assume that n 1. Prove
that
if
F
(n)
=
d|n f (d), then f (n) =
d|n (d)F (n/d).
P
Solution. Assume F (n) = d|n f (d). Then
X

(d)F (n/d) =

d|n

(d)

d|n

X
d |

XX

f (d )

n
d

(d)f (d )

d |n d| n
d

(n/d )f (d )

d |n

= f (n).
P
P
(b) Assume that n 1. Prove
that
if
f
(n)
=
(n/d)F
(d),
then
F
(n)
=
d|n
d|n f (d).
P
Solution. Assume f (n) = d|n (n/d)F (d). Then
X

f (d) =

XX

(d/d )F (d )

XX

(d )F (d ),

d|n

d|n d |d

changing variable: d = d d

d |n d | n
d

(n/d )F (d )

d |n

= F (n).

Keeping Scores:
Question Bonus
Score
Out of
1

Total

10

10

13

11

58

Question 2. (4+3+3 points) (a) Let f, g be arithmetic functions. Prove that


Df g (s) = Df (s)Dg (s).
Solution. We have
Df g (s) =

ns

n1

f (d)g(n/d),

using n = dd

d|n

XX

(dd )s f (d)g(d )

d1 d 1

= Df (s)Dg (s).
(b) Show that the Dirichlet series Dk of k is
Dk (s) = (s k).
Solution. Observe that
Dk (s) =

ns k (n) =

n1

ns nk =

n1

n(sk) = (s k).

n1

(c) Determine the Dirichlet series Dk .


Solution. We note first that
X
X
k (n) =
dk 1 =
k (d)0 (n/d) = (k 0 )(n).
d|n

d|n

Hence, we have from parts (a) and (b) that


Dk (s) = Dk (s)D0 (s) = (s k)(s).

QuestionP3. (4+4 points) (a) Let f be a multiplicative function with f (1) = 1. Define
F (n) = d|n (d)f (d). Prove that
Y
F (n) =
(1 f (p)).
p|n

P
Solution. Let F (n) = d|n (d)f (d). Consider F (pa ), where p is prime and a is a positive
integer. Then since (1) = 1, (p) = 1 and (pi ) = 0 for all i 2, we have
a

F (p ) =

X
d|pa

(d)f (d) =

a
X

(pi )f (pi ) = (1)f (1) + (p)f (p) = 1 f (p).

i=0

Also, since both and f are multiplicative functions, the function F (n) =
also multiplicative. Thus
Y
Y
F (n) =
F (pep ) =
(1 f (p)).
p|n

p|n

d|n

(d)f (d) is

P
(b) Let F be the arithmetic function given by F (n) = d|n (d) (d). Show that F (n) =
(1)(n) .
Solution. We apply part (a), with f = . We note that for prime p, we have (p) = 2. Then
for n 1, we have
Y
F (n) =
(1 (p)) = (1)(n) .
p|n

(5 points) The CRT says that for a, b Z, x a (mod 11) and x b


Question 4.
(mod 13) iff x 78a 77b (mod 143). Find all x so that x2 126 (mod 143).
Solution. Note that x2 126 (mod 143) is equivalent to
x2 126 5 16

(mod 11),

and

x 126 9

(mod 13).

The above is equivalent to


x 4

(mod 11),

and

x 3

(mod 13).

The CRT now gives


x 81,

x 114

(mod 143).


Question 5. (4+5+4 points) (a) Let p = 131 which is prime and a = 59. Compute ap .
Solution. There are many ways to compute the value. For example,
 
 
 
 
 
 
 
131
13
59
7
6
1
59
=
=
=
=
=
=
= 1.
131
59
59
13
13
7
7

(b) Let p be prime satisfying p 3 (mod 4) and a Z satisfying ap = 1. Show that
x a(p+1)/4 is a square
root of a modulo p.

Solution. Since ap = 1, we have from Eulers Criterion that
 
a
1=
a(p1)/2
p

(mod p).

Thus, for x a(p+1)/4 ,


x2 a(p+1)/2 = a a(p1)/2 a (mod p).
(c) Find all incongruent square roots of a = 59 modulo p = 131. (Note that a32 46
(mod p)).
Solution. From (b), we see that
x a(p+1)/4 = a33 = 46 59 94

(mod p),

is a square root of 59 modulo p. Since p = 131 is prime and a is a quadratic residue, there
are two incongruent square roots of 59 modulo p: 94.

Question 6. (2+5+4 points) (a) State carefully Eulers Criterion.


Solution. Assume that p is an odd prime and a is an integer with p a. Then
(mod p).
(b) Prove Eulers Criterion.
Solution. There are two cases. (I) If
and p x (because p a). Then
a(p1)/2 x2

a
p

a
p

a(p1)/2

= 1, then there is x Z so that x2 a (mod p)

(p1)/2

= xp1 1

(mod p),

where the last congruence follows from FLT.


(II) If ap = 1, then there does not exist x so that x2 a (mod p). We note first that
for each x satisfying 1 x p 1, there is y ax (mod p) so that xy a (mod p).
As a is a quadratic nonresidue, we see that x 6 y (mod p). This means that the integers
1, 2, . . . , p 1 can be paired as {x, y} so that xy a (mod p). (To prove this rigorously, one
would want to show that the mapping f : [p 1] [p 1] defined by f (x) = ax mod p
satisfies f (x) 6= x and f (f (x)) = x for all x [p 1].) Then
(1)(2) (p 1) a(p1)/2

(mod p).

Wilson Theorem gives that (p 1)! 1 (mod p), and hence a(p1)/2 1 (mod p).

(c) Assume that p is an odd prime. Prove that 1
= 1 iff p 1 (mod 4).
p
Solution. Since p is an odd prime, we see that p 1, 3 (mod 4). If p = 4k + 1 for some k,
then
 
1
(1)(p1)/2 = (1)2k = 1 (mod p).
p


1
Since 1
=
1
and
p

3,
we
see
that
= 1 in this case. If p = 4k + 3 for some k, then
p
p


and we have

1
p

1
p

(1)(p1)/2 = (1)2k+1 = 1

= 1 in this case.

(mod p),

The following notation and results can be useful in this exam.


Q
We use n = p|n pep to represent the prime power factorization of n.

The functions , k , k , where k 0, and are defined as follows. (a)P(n) = 1 if


n = 1, and (n) = 0 if n 2. (b) k (n) = nk for n Z+ . (c) k (n) = d|n dk . (d)
= 0 . (e) (n) equals the number of distinct prime divisors of n for n 1.
P
The Mobius function is , and it satisfies d|n (d) = (n).
The Dirichlet convolution of arithmetic functions f and g is the function f g given
by
X
(f g)(n) =
f (d)g(n/d),
n 1.
d|n

The Dirichlet series of an arithmetic function f is Df and is given by


X
Df (s) =
ns f (n),
for values of s whenever the series is convergent.
n1

Let f, F be arithmetic functions satisfying F (n) =


multiplicative iff f is multiplicative.

d|n

f (d). It is known that F is

You might also like